top of page

Register for the PLAB 1 Question Bank

Pelvic X-Ray

>400 QUESTIONS 3 X MOCKS $3

Select your option for subscription below. You will then have access to our exclusive PLAB question bank, tailored to the UK MLA. 
 

PLAB Part 1

PLAB 1 is a written exam, consisting of 180 multiple choice questions that you must answer over 3 hours. Each question starts with a small scenario followed by a question. You need to pick the right answer out of five options. We will give you access to our question bank covering all PLAB 1 topics. This includes medical ethics and UK medical law, which we know our international candidates struggle with. 

​

We also explain why the answer is correct, to help you really improve your PLAB 1 knowledge.



Unlimited question bank access = £2.99  

(DISCOUNT)

Click HERE to access. â€‹â€‹

​

EXAMPLE QUESTIONS:

​​

1. Cardiology

1. A 65-year-old man presents with sudden onset severe chest pain, which he describes as tearing in nature, radiating to his back. He has a history of hypertension and smoking. On examination, his blood pressure is 180/100 mmHg, with a pulse rate of 95 bpm. His physical exam reveals a difference in blood pressure between his left and right arms, and a mid-systolic murmur is heard at the left sternal border.

Which is the most appropriate diagnostic investigation?

A) Chest X-ray
B) CT aortogram
C) Echocardiography
D) ECG
E) D-dimer

​

​

2. Respiratory Medicine

2. A 72-year-old man with a 40-pack-year smoking history presents with worsening dyspnea, a chronic cough, and sputum production over the last 3 months. He also reports recent weight loss and a new onset of haemoptysis. On examination, he has decreased breath sounds over the right lower lobe and clubbing of the fingers. A chest X-ray reveals a mass in the right lung with evidence of collapse in the lower lobe.

What is the most likely diagnosis, and what is the next best investigation?

A) Tuberculosis, sputum culture for acid-fast bacilli
B) Lung cancer, CT chest
C) Pneumonia, sputum culture
D) Chronic obstructive pulmonary disease, spirometry
E) Pulmonary embolism, CT pulmonary angiography

​

​

3. Rheumatology

3. A 45-year-old woman presents with a 2-month history of progressive fatigue, morning stiffness, and pain in her small joints, particularly in the wrists and metacarpophalangeal joints. The symptoms are worse in the morning and last for over an hour. She also has difficulty opening jars and has noticed swelling in the joints. Her family history is significant for rheumatoid arthritis.

Which is the most likely diagnosis?

A) Osteoarthritis
B) Rheumatoid arthritis
C) Psoriatic arthritis
D) Gout
E) Systemic lupus erythematosus

​

​​

4. Endocrinology

4. A 40-year-old woman presents with weight gain, fatigue, and constipation over the last 6 months. She also has a hoarse voice and has noticed that her skin has become more dry. On examination, she has periorbital puffiness and delayed reflexes. Blood tests show a TSH of 18.5 mU/L, free T4 of 6.5 pmol/L, and elevated anti-thyroid peroxidase (TPO) antibodies.

What is the most likely diagnosis?

A) Hashimoto's thyroiditis
B) Graves' disease
C) Subclinical hypothyroidism
D) Pituitary adenoma
E) Thyroid cancer

​

5. Medical Ethics

5. A 40-year-old woman is brought to the emergency department with a life-threatening anaphylactic reaction. Her husband is present but refuses to consent to life-saving treatment because of his religious beliefs. What is the most appropriate course of action?

A) Respect the husband’s decision and withhold treatment
B) Proceed with life-saving treatment despite the husband’s objections, as the patient’s life is at risk
C) Delay treatment until a court order is obtained
D) Seek an ethics consultation to mediate the situation
E) Request the patient’s consent, even though she is confused and in extremis

​

Answers and Explanations:

  1. B) CT aortogram

    • Explanation: The presentation of sudden, severe, tearing chest pain radiating to the back, along with a history of hypertension, suggests a type A aortic dissection. The most appropriate investigation to confirm this diagnosis is a CT aortogram, which is highly sensitive for detecting aortic dissection.

    • Why the others are incorrect:

      • A) Chest X-ray: A chest X-ray may show widening of the mediastinum but is not diagnostic.

      • C) Echocardiography: While echocardiography is useful for detecting aortic dissection in some cases, CT angiography is preferred for precise diagnosis.

      • D) ECG: An ECG is used to rule out ischemic causes of chest pain, but this patient’s presentation points to aortic dissection rather than MI.

      • E) D-dimer: D-dimer is typically used to rule out venous thromboembolism, but it’s not specific for diagnosing aortic dissection.

  2. B) Lung cancer, CT chest

    • Explanation: The patient's smoking history, weight loss, hemoptysis, and the mass on chest X-ray point to lung cancer, specifically non-small cell lung cancer (NSCLC). A CT chest is the next best investigation to further characterize the mass and assess for metastases.

    • Why the others are incorrect:

      • A) Tuberculosis: While tuberculosis can cause weight loss and hemoptysis, the mass on chest X-ray is more indicative of cancer.

      • C) Pneumonia: Pneumonia would typically present with more acute symptoms like fever and productive cough, and chest X-ray would show consolidation rather than a mass.

      • D) COPD: This is unlikely given the hemoptysis and the presentation of a mass.

      • E) Pulmonary embolism: PE would not present with a mass on chest X-ray; it usually presents with sudden onset dyspnea and pleuritic chest pain.

  3. B) Rheumatoid arthritis

    • Explanation: The patient presents with symmetric joint pain, morning stiffness lasting more than an hour, and swelling in the hands and wrists. These are characteristic features of rheumatoid arthritis (RA). The family history also supports this diagnosis.

    • Why the others are incorrect:

      • A) Osteoarthritis: OA typically causes joint pain that worsens with activity and is not associated with prolonged morning stiffness.

      • C) Psoriatic arthritis: Psoriatic arthritis often affects the distal joints and is associated with psoriasis, which this patient does not have.

      • D) Gout: Gout typically affects the big toe and is not associated with symmetric joint involvement.

      • E) SLE: While SLE can present with joint pain, it is more likely to involve other systemic manifestations such as a characteristic rash and renal involvement.

  4. A) Hashimoto's thyroiditis

    • Explanation: The patient presents with fatigue, weight gain, and cold intolerance, and laboratory tests show elevated TSH, low T4, and elevated anti-TPO antibodies, which are consistent with Hashimoto's thyroiditis, an autoimmune cause of hypothyroidism.

    • Why the others are incorrect:

      • B) Graves' disease: Graves’ disease causes hyperthyroidism, not hypothyroidism.

      • C) Subclinical hypothyroidism: Subclinical hypothyroidism would have a normal T4 level, whereas this patient has low T4.

      • D) Pituitary adenoma: Pituitary adenomas typically cause secondary hypothyroidism, but this patient's autoimmune markers suggest primary hypothyroidism.

      • E) Thyroid cancer: Thyroid cancer can cause hypothyroidism in some cases, but there is no mention of a thyroid nodule, and the autoimmune markers point to Hashimoto’s thyroiditis.

  5. B) Proceed with life-saving treatment despite the husband's objections, as the patients life is at risk. 

    • Explanation: In an emergency situation where the patient's life is at imminent risk, the patient’s right to life and the healthcare team’s duty to preserve life override any objections from family members, including the husband, even if those objections are based on religious beliefs. Implied consent applies in emergencies when the patient is unconscious or unable to consent. This principle allows medical professionals to provide necessary life-saving treatment without the need for explicit consent if waiting for consent would result in harm.

    • Why the others are incorrect:

      • A) Respect the husband’s decision and withhold treatment

        Incorrect: The patient’s life is at immediate risk, and the husband's objections cannot override the need for life-saving treatment. Withholding treatment could lead to death.

      • C) Delay treatment until a court order is obtained

        Incorrect: Delaying life-saving treatment to obtain a court order is dangerous and could result in the patient’s death. Immediate action is required in emergencies.

      • D) Seek an ethics consultation to mediate the situation

        Incorrect: An ethics consultation is unnecessary in an emergency where time is critical. Immediate treatment must be provided to save the patient’s life.

      • E) Request the patient’s consent, even though she is in extremis and confused

        Incorrect: The patient is unable to provide legal consent due to the severity of the anaphylactic reaction. Implied consent allows for immediate treatment in such cases.

​

​

​

bottom of page